Page 1 of 1

The Mechanistic Approach for Assumption Question

Posted: Fri Oct 27, 2017 6:34 pm
by utpalbarman
Hi PowerScore Folks,
I am struggling with Assumption questions and trying to find an approach that will help me. I was again reading the PS Justify the Conclusion chapter and found " The Mechanistic Approach ( aka TMA)" and was wondering if i can rely on it for Assumption Questions since both of them belong to Second Help family and how much should i put the effort to mastery it since by reading the " Mechanistic Approach" it sound like i can apply this approach not only to Justify also Assumption and Strengthen Question.

If i blindly use the Assumption Negation technique i am not able to identify the answer correctly. I am planning to do below as an strategy to be successful in attacking Assumption questions;

In my understanding any argument in the Help family can be generalized below:

1) Unstated Premise ( This Premise is never present in the argument)
2) Premise 1
3) Premise 2
3)Conclusion

Conceptually i know that for strengthen answer choice i need to find an answer choice that proves the conclusion from range of 1% to 100% , Justify Answer should be 100% true for Conclusion to be valid.
Now how much i can rely on the Mechanistic Approach to find the Unstated Premise?
Also the book mention that the approach can be used for Assumption question but not always.
Can you folks please clarify when TMA approach should be used for assumption and when not?
Thanks,
Utpal

Re: The Mechanistic Approach for Assumption Question

Posted: Sat Oct 28, 2017 5:06 pm
by Dave Killoran
utpalbarman wrote:Hi PowerScore Folks,
I am struggling with Assumption questions and trying to find an approach that will help me. I was again reading the PS Justify the Conclusion chapter and found " The Mechanistic Approach ( aka TMA)" and was wondering if i can rely on it for Assumption Questions since both of them belong to Second Help family and how much should i put the effort to mastery it since by reading the " Mechanistic Approach" it sound like i can apply this approach not only to Justify also Assumption and Strengthen Question.

If i blindly use the Assumption Negation technique i am not able to identify the answer correctly. I am planning to do below as an strategy to be successful in attacking Assumption questions;

In my understanding any argument in the Help family can be generalized below:

1) Unstated Premise ( This Premise is never present in the argument)
2) Premise 1
3) Premise 2
3)Conclusion

Conceptually i know that for strengthen answer choice i need to find an answer choice that proves the conclusion from range of 1% to 100% , Justify Answer should be 100% true for Conclusion to be valid.
Now how much i can rely on the Mechanistic Approach to find the Unstated Premise?
Also the book mention that the approach can be used for Assumption question but not always.
Can you folks please clarify when TMA approach should be used for assumption and when not?
Thanks,
Utpal
Hi U,

Thanks for the question! Let's talk a little bit about Assumption strategy in general and then about the Mechanistic Approach.

First, you mentioned that, "If i blindly use the Assumption Negation technique i am not able to identify the answer correctly." One point I want to make is that you should never use a technique blindly, or apply it to all five answers choices (which is what I fear you are doing). It takes too much time to use this technique on every answer, so I try to make it very clear in the books and our courses that the Assumption Negation Technique should not be used on every answer, but only on the one or two answers that are Contenders. That makes the technique very powerful as a double-check/separating tool while also not requiring a massive amount of time to apply. This also means that this is just a tool to be used in certain instances; it's not a standard approach to solving Assumption questions. In other words, this only comes in at the end, and isn't a go-to strategy when you are initially attacking these questions. Does that make sense?

The first step in any attack is, as you rightly noted, to break down the argument into component parts. This is really an interpretative step, because you are attempting to do two things: understand the argument and reduce it into simple terms, and to identify the major pieces of the argument (such as the premises, conclusion etc). I would not agree that arguments are generally two premises and a conclusion, however. Many are, but there are so many other structures that there's no reason to generalize here.

As far as your understanding of Strengthen and Justify in numerical terms, your assessment is correct. Good job! I would ask, though, does the way those question types work make sense to you? For example, knowing Strengthen is 1% to 100% and Justify is 100% is good in the abstract, but it's far more important to grasp what those mean in direct, operational terms, as well as why they work that way. I'd emphasize that understanding what's actually happening is the key point here, so make sure to study that carefully.

And now to your question about the Mechanistic Approach and Assumption questions. First, will it work sometimes? Yes, especially in Supporter Assumptions. They work very much the same as Justify questions (which is why you see answers in Justify questions that are also assumptions of the argument). Even so, as I say in the books, we don't recommend using the Mechanistic Approach to specifically attack Assumption questions, and the reason is simple: it won't always work. Especially in Defender Assumptions instances, the Mechanistic Approach won't reveal the correct answer or even take you near it. So, because it's not 100% reliable for Assumption questions, we don't advocate using it. As an aside, because it is 100% reliable (when applied properly) in Justify questions, that's why you see us limit the technique to those types.

So, that last paragraph is a tricky one, so I want to distill the message: the Mechanistic Approach can at times help in Assumption questions, but you don't know ahead of time when it will, so we can't recommend it as a standard tool in Assumption questions. It always works in Justify questions, so it can be used there in any instance.

Please let me know if that helps. Thanks!